Đến nội dung

Hình ảnh

Bất Đẳng Thức Qua Các Kỳ TS ĐH


  • Please log in to reply
Chủ đề này có 208 trả lời

#181
Ispectorgadget

Ispectorgadget

    Nothing

  • Quản lý Toán Phổ thông
  • 2946 Bài viết
Bài 83: Cho x,y,z là các số thực dương . Tìm GTNN của biểu thức
$P = \sqrt[3]{{4(x^3 +y^3 )}}+\sqrt[3]{{4(x^3 +z^3 )}}+\sqrt[3]{{4(z^3+y^3 )}}+2(\frac{x}{{y^2 }}+\frac{y}{{z^2 }}+\frac{z}{{x^2 }})$

Dự bị khối A năm 2007
Bài 84: Cho a,b>0 thỏa mãn $ab+b+a=3$. Chứng minh rằng
$\frac{{3a}}{{b+1}}+\frac{{3b}}{{a+1}}+\frac{{ab}}{{a + b}} \le a^2 +b^2 +\frac{3}{2}$
Dự bị khối D -2007

Bài viết đã được chỉnh sửa nội dung bởi Ispectorgadget: 11-03-2012 - 13:16

►|| The aim of life is self-development. To realize one's nature perfectly - that is what each of us is here for. ™ ♫


#182
tieulyly1995

tieulyly1995

    Sĩ quan

  • Thành viên
  • 435 Bài viết

Bài 83: Cho x,y,z là các số thực dương . Tìm GTNN của biểu thức
$P = \sqrt[3]{{4(x^3 +y^3 )}}+\sqrt[3]{{4(x^3 +z^3 )}}+\sqrt[3]{{4(z^3+y^3 )}}+2(\frac{x}{{y^2 }}+\frac{y}{{z^2 }}+\frac{z}{{x^2 }})$

Dự bị khối A năm 2007
Bài 84: Cho a,b>0 thỏa mãn $ab+bc+ac=3$. Chứng minh rằng
$\frac{{3a}}{{b+1}}+\frac{{3b}}{{a+1}}+\frac{{ab}}{{a + b}} \le a^2 +b^2 +\frac{3}{2}$
Dự bị khối D -2007

Bài 83 :
Ta có :
$4( x^{3}+y^{3})\geq (x+y)^{3}$ ( c/m bằng phương pháp biến đổi tương đương )
tương tự :
$4( y^{3}+z^{3})\geq (y+z)^{3}$
$4( z^{3}+x^{3})\geq (z+x)^{3}$
Do đó :
$\sqrt[3]{4(x^{3}+y^{3})}+ \sqrt[3]{4(y^{3}+z^{3})}+\sqrt[3]{4(z^{3}+y^{3})}\geq 2(x+y+z)\geq 6\sqrt[3]{xyz}$
Mặt khác :
$2(\frac{x}{y^{2}}+\frac{y}{z^{2}}+\frac{z}{x^{2}})\geq \frac{6}{\sqrt[3]{xyz}}$
Suy ra :
$P\geq 6\sqrt[3]{xyz}+\frac{6}{\sqrt[3]{xyz}}\geq 12$
Vậy min $P=12\Leftrightarrow \left\{\begin{matrix} xyz=1 & \\x=y=z & \end{matrix}\right.\Leftrightarrow x=y=z=1$
p/s : bài 84 e chép đề sai rồi :closedeyes: . Phần gt có chứa c mà phần c/m chẳng đề cập đến :wub:

#183
tieulyly1995

tieulyly1995

    Sĩ quan

  • Thành viên
  • 435 Bài viết

Bài 84: Cho a,b>0 thỏa mãn $ab+b+a=3$. Chứng minh rằng
$\frac{{3a}}{{b+1}}+\frac{{3b}}{{a+1}}+\frac{{ab}}{{a + b}} \le a^2 +b^2 +\frac{3}{2}$
Dự bị khối D -2007

Từ $ab+b+a=3$ suy ra :
$\left\{\begin{matrix} ab=3-(a+b) & \\ (a+1)(b+1)=4 & \end{matrix}\right.$
Biến đổi tương đương
$BĐT \Leftrightarrow \frac{3a(a+1)+3b(b+1)}{(a+1)(b+1)}+\frac{3}{a+b}-1\leq a^{2} +b^{2}+\frac{3}{2}$

$\Leftrightarrow \frac{3(a^{2}+b^{2}+a+b)}{4}+\frac{3}{a+b}-1\leq a^{2}+b^{2}+\frac{3}{2}$

$\Leftrightarrow (a^{2}+b^{2})-3(a+b)-\frac{12}{a+b}+10\geq 0$ ( * )

Đặt $t=a+b$ , (do $a,b> 0\Rightarrow t> 0$)
$\Rightarrow t^{2}=(a+b)^{2}\geq 4ab= 4(3-(a+b))=4(3-t)$
$\Rightarrow t^{2}+4t-12\geq 0\Rightarrow t\geq 2$ (chú ý $t>0$)
Mặt khác :
$a^{2}+b^{2}=t^{2}-2ab= t^{2}-2(3-t)=t^{2}+2t-6$

$( * )\Leftrightarrow t^{2}-t-\frac{12}{t}+4\geq 0, t\geq 2$


$\Leftrightarrow t^{3}-t^{2}+4t-12\geq 0,t\geq 2$

$\Leftrightarrow (t-2)(t^{2}+t+6)\geq 0, t\geq 2$ (luôn đúng )

$\Leftrightarrow $đpcm

Bài viết đã được chỉnh sửa nội dung bởi Ispectorgadget: 11-03-2012 - 13:53


#184
dark templar

dark templar

    Kael-Invoker

  • Hiệp sỹ
  • 3788 Bài viết
Góp vui 1 bài :D
Bài 85: Cho $x,y,z>0$ thỏa mãn:$xyz=1$.Tìm GTNN của:
$$P=\frac{1}{(1+x)^{2012}}+\frac{1}{(1+y)^{2012}}+\frac{1}{(1+z)^{2012}}$$
(Thi thử ĐH khối A trường Chuyên Lê Hồng Phong TPHCM)
"Do you still... believe in me ?" Sarah Kerrigan asked Jim Raynor - Starcraft II:Heart Of The Swarm.

#185
alex_hoang

alex_hoang

    Thượng úy

  • Hiệp sỹ
  • 1152 Bài viết

Góp vui 1 bài :D
Bài 85: Cho $x,y,z>0$ thỏa mãn:$xyz=1$.Tìm GTNN của:
$$P=\frac{1}{(1+x)^{2012}}+\frac{1}{(1+y)^{2012}}+\frac{1}{(1+z)^{2012}}$$
(Thi thử ĐH khối A trường Chuyên Lê Hồng Phong TPHCM)

Bài này có vẻ là suy từ bài toán sau
Cho $a,b,c>0$ thoả mãn $abc=1$ thì
$$ \frac{1}{(a+1)^2}+\frac{1}{(b+1)^2}+\frac{1}{(c+1)^2} \ge \frac{3}{4}$$

Bài viết đã được chỉnh sửa nội dung bởi alex_hoang: 11-03-2012 - 17:56

alex_hoang


HẸN NGÀY TRỞ LẠI VMF THÂN MẾN

http://www.scribd.co...oi-Ban-Cung-The

#186
alex_hoang

alex_hoang

    Thượng úy

  • Hiệp sỹ
  • 1152 Bài viết
Bài 86 Cho các số thực dương $x,y,z$ thỏa mãn $4(x+y+z)=3xyz$. Tìm giá trị lớn nhất của biểu thức:
$$P=\dfrac{1}{2+x+yz}+\dfrac{1}{2+y+zx}+\dfrac{1}{2+z+xy}$$

(Đề thi thử trương Đại Học Vinh)


alex_hoang


HẸN NGÀY TRỞ LẠI VMF THÂN MẾN

http://www.scribd.co...oi-Ban-Cung-The

#187
alex_hoang

alex_hoang

    Thượng úy

  • Hiệp sỹ
  • 1152 Bài viết
Bài 87:Cho $x,\,y,\,z$ là các số thực dương thỏa mãn $xyz=x+y+z+2.$ Chứng minh rằng $$\sqrt{x}+\sqrt{y}+\sqrt{z} \le \frac{3}{2}\sqrt{xyz}.$$

Trích đề thi trường Trần Quốc Tuấn lần 3 2012


alex_hoang


HẸN NGÀY TRỞ LẠI VMF THÂN MẾN

http://www.scribd.co...oi-Ban-Cung-The

#188
whiterose96

whiterose96

    Hạ sĩ

  • Thành viên
  • 80 Bài viết

Bài 82: Cho 2 số thực dương x,y thay đổi thỏa mãn $3x+y\le1$. Tìm GTNN của biểu thức.
$$A=\frac{1}{x}+\frac{1}{\sqrt{xy}}$$

Đề thi Cao đẳng năm 2010


Áp dụng AM-GM
$\frac{x+y}{2}\geq \sqrt{xy}\Rightarrow \frac{1}{\sqrt{xy}}\geq \frac{2}{x+y}$
$\Rightarrow A\geq \frac{1}{x}+\frac{2}{x+y}\geq 2\sqrt{\frac{2}{x(x+y))}} \Rightarrow A\geq \frac{4}{\sqrt{2x\left ( x+y \right )}}\geq \frac{8}{2x+x+y}\geq 8$
đẳng thức xảy ra khi $x=y=\frac{1}{4}$

Hình đã gửi


#189
whiterose96

whiterose96

    Hạ sĩ

  • Thành viên
  • 80 Bài viết

Bài 65: Cho x,y,z là 3 số thực dương. Tìm GTLN của
$P=\dfrac{x}{x+\sqrt{(x+y)(x+z)}}+\dfrac{y}{y+\sqrt{(y+z)(y+x)}}+\dfrac{z}{z+\sqrt{(y+z)(z+x)}}$


áp dụng Cauchy-Schwarz:
${(x+y)(x+z)}\geq \left ( \sqrt{xy} +\sqrt{xz}\right )^{2}$
$\Rightarrow \sqrt{{(x+y)(x+z)}}\geq \left ( \sqrt{xy} +\sqrt{xz}\right )$
$\Rightarrow \frac{x}{x+\sqrt{(x+y)(x+z)}}\leq \frac{\sqrt{x}}{\sqrt{x}+\sqrt{y}+\sqrt{z}}$
Tương tự
$\Rightarrow P\leq 1$
ĐTXR khi x=y=z

Hình đã gửi


#190
Ispectorgadget

Ispectorgadget

    Nothing

  • Quản lý Toán Phổ thông
  • 2946 Bài viết

Bài 86 Cho các số thực dương $x,y,z$ thỏa mãn $4(x+y+z)=3xyz$. Tìm giá trị lớn nhất của biểu thức:
$$P=\dfrac{1}{2+x+yz}+\dfrac{1}{2+y+zx}+\dfrac{1}{2+z+xy}$$

(Đề thi thử trương Đại Học Vinh)

Theo BĐT $Cauchy-Schwarz$ thì ta có:
$$4P \le \sum_{cyc} \frac{1}{x+2} + ( \frac{1}{xy} + \frac{1}{yz}+ \frac{1}{zx} ) = \sum_{cyc}\frac{1}{x+2} + \frac34$$
Ta sẽ chứng minh:
$$\sum_{cyc} \frac{1}{x+2} \le \frac34 \Leftrightarrow xy + yz +zx \ge 12$$
Nhưng mặt khác, theo BĐT $Cauchy-Schwarz$ thì:
$$ ( xy + yz + zx) ( \frac{1}{xy} + \frac{1}{yz}+ \frac{1}{zx} ) \ge 9 \Leftrightarrow xy + yz + zx \ge 12$$
Suy ra $Max P = \frac38 $.
Dấu "=" xảy ra khi và chỉ khi: $ x= y =z =2$

Nguồn: http://boxmath.vn/4r...1-2-z-xy-25513/

►|| The aim of life is self-development. To realize one's nature perfectly - that is what each of us is here for. ™ ♫


#191
Tham Lang

Tham Lang

    Thượng úy

  • Thành viên
  • 1149 Bài viết

Bài 81(Đề thi thử đại học số 10-Boxmath.vn)
Cho $x,y,z$ là các số thực thuộc $[0,1]$.Tìm giá trị nhỏ nhất của biểu thức
$$P=(xy-y+1)^2+(yz-z+1)^2+(zx-x+1)^2$$

$$ P = (x(y - 1))^2 + (y(y - 1))^2 + (z(z - 1))^2 + 2\left ((x(y - 1) + y(z - 1) + z(x - 1) \right ) + 3 \ge 2\left (x(y - 1) + y(z - 1) + z(x - 1) \right ) + 3$$
Lại có $$(x - 1)(y - 1)(z - 1) \le 0 \Leftrightarrow xyz - (xy + yz + xz - x - y - z) - 1 \le 0 \Leftrightarrow xy + yz + zx - x - y - z \ge xyz - 1 \ge -1$$
Do đó $$P \ge 3 - 2 = 1$$
Đẳng thức xảy ra khi 2 trong 3 số = 1, số còn lại = 0

Off vĩnh viễn ! Không ngày trở lại.......


#192
Tham Lang

Tham Lang

    Thượng úy

  • Thành viên
  • 1149 Bài viết

Bài 73(Đề thi thử đại học số 2-Boxmath.vn)
Cho các số thực $a,b,c$ đôi một khác nhau.Chứng minh rằng:

\[{\left( {1 + \frac{{a - 2b}}{{b - c}}} \right)^2} + {\left( {1 + \frac{{b - 2c}}{{c - a}}} \right)^2} + {\left( {1 + \frac{{c - 2a}}{{a - b}}} \right)^2} \ge 8\]


$$VT = \left (\dfrac{b + c - a}{b - c}\right )^2 + \left (\dfrac{c + a - b}{c - a} \right )^2 + \left (\dfrac{a + b - c}{a - b}\right )^2$$
Lúc đó đặt $$x = b + c - a, y = c + a - b, z = a + b - c \Leftrightarrow a = \dfrac{y + z}{2}, b= \dfrac{x + z}{2}, c = \dfrac{x + y}{2}$$
$$\Leftrightarrow b - c = \dfrac{z - y}{2}, c - a = \dfrac{x - z}{2}, a - b = \dfrac{y - x}{2}$$
$$VT = \dfrac{4.x^2}{(y - z)^2} + \dfrac{4y^2}{(x - z)^2} + \dfrac{4z^2}{(x - y)^2}$$
Đặt $m = \dfrac{x}{y - z}, n = \dfrac{y}{z - x}, p = \dfrac{z}{x - y}$
Thì $$(m + 1)(n + 1)(p + 1) = (m - 1)(n - 1)(z - 1) \Leftrightarrow mn + np + pm = -1$$
do đó
$$ P = 4\left (m^2 + n^2 + p^2\right ) \ge -4.2.\left (mn + np + pm \right ) = 8$$
Suy ra ĐPCM.

Bài viết đã được chỉnh sửa nội dung bởi huymit_95: 12-03-2012 - 14:42

Off vĩnh viễn ! Không ngày trở lại.......


#193
Ispectorgadget

Ispectorgadget

    Nothing

  • Quản lý Toán Phổ thông
  • 2946 Bài viết

Bài 87:Cho $x,\,y,\,z$ là các số thực dương thỏa mãn $xyz=x+y+z+2.$ Chứng minh rằng $$\sqrt{x}+\sqrt{y}+\sqrt{z} \le \frac{3}{2}\sqrt{xyz}.$$

Trích đề thi trường Trần Quốc Tuấn lần 3 2012

Từ giả thiết suy ra :
$$\frac{1}{1+x}+\frac{1}{1+y}+\frac{1}{1+z}=1$$
Đặt $a=\frac{1}{1+x};b=\frac{1}{1+y};c=\frac{1}{1+z}$
Suy ra: $$\Rightarrow x=\frac{1-a}{a}=\frac{b+c}{a};y=\frac{c+a}{b};z=\frac{a+b}{c}$$
Điều cần chứng minh tương đương:
$$\sqrt{x}+\sqrt{y}+\sqrt{z}\leq \frac{3}{2}\sqrt{xyz}\Leftrightarrow \sqrt{\frac{a}{b+c}.\frac{b}{a+c}}+\sqrt{\frac{b}{c+a}.\frac{c}{a+b}}+\sqrt{\frac{c}{a+b}.\frac{a}{b+c}}\leq \frac{3}{2}$$
Áp dụng BĐT AM-GM ta có
$$\sqrt{\frac{a}{b+c}.\frac{b}{c+a}}\leq \frac{1}{2}(\frac{a}{a+c}+\frac{b}{b+c})$$
Chứng minh tương tự rồi cộng lại ta có điều cần chứng minh.

►|| The aim of life is self-development. To realize one's nature perfectly - that is what each of us is here for. ™ ♫


#194
alex_hoang

alex_hoang

    Thượng úy

  • Hiệp sỹ
  • 1152 Bài viết
Bài 88Chứng minh rằng với mọi số thực dương $a,b,c$.Chứng minh rằng:
$$\frac{1}{{2{a^2} + bc}} + \frac{1}{{2{b^2} + ca}} + \frac{1}{{2{c^2} + ab}} \le {\left( {\frac{{a + b + c}}{{ab + bc + ca}}} \right)^2}$$
Đề thi thử ĐH THPT Chuyên Thái Bình
alex_hoang


HẸN NGÀY TRỞ LẠI VMF THÂN MẾN

http://www.scribd.co...oi-Ban-Cung-The

#195
Ispectorgadget

Ispectorgadget

    Nothing

  • Quản lý Toán Phổ thông
  • 2946 Bài viết

Góp vui 1 bài :D
Bài 85: Cho $x,y,z>0$ thỏa mãn:$xyz=1$.Tìm GTNN của:
$$P=\frac{1}{(1+x)^{2012}}+\frac{1}{(1+y)^{2012}}+\frac{1}{(1+z)^{2012}}$$
(Thi thử ĐH khối A trường Chuyên Lê Hồng Phong TPHCM)

:) bài này khá hay
Trước hết ta chứng minh bất đẳng thức sau $\frac{1}{(1+x)^2}+\frac{1}{(1+y)^2}+\frac{1}{(1+z)^2}\geq \frac{3}{4}$
Áp dụng bất đẳng thức Cauchy-Schwarz ta có:
$$\frac{1}{(1+x)^2}+\frac{1}{(1+y)^2}\geq \frac{1}{(1+\frac{x}{y})(1+xy)}+\frac{1}{(1+\frac{y}{x})(1+xy)}=\frac{1}{1+xy}=\frac{z}{z+1}$$
Từ đó suy ra: $$\frac{1}{(1+x)^2}+\frac{1}{(1+y)^2}+\frac{1}{(1+z)^2}\geq \frac{z}{z+1}+\frac{1}{(z+1)^2}=\frac{z^2+z+1}{(z+1)^2}=\frac{3}{4}+\frac{(z-1)^2}{4(z+1)^2}\geq \frac{3}{4}$$
Bất đẳng thức trên được chứng minh.
Để đơn giản ta đặt $a=\frac{1}{(1+x)^2};b=\frac{1}{(1+y)^2};c=\frac{1}{(1+z)^2}$
Ta quy về tìm giá trị nhỏ nhất của $a^{1006}+b^{1006}+c^{1006}$ với $a+b+c\geq \frac{3}{4}$
Áp dụng bất đẳng thức AM-GM cho 1006 số không âm ta có:
$$a^{1006} + \underbrace {\frac{1}{{4^{1006} }} + \frac{1}{{4^{1006} }} + ... + \frac{1}{{4^{1006} }}}_{1005times} \ge 1006\sqrt[{1006}]{{a^{1006} .\frac{1}{{4^{10110300} }}}} = 1006a.\frac{1}{{4^{1005} }}$$
Tương tự với $b,c$. Từ đây ta suy ra: $$a^{1006}+b^{1006}+c^{1006}\geq \frac{1006}{4^{1005}}(a+b+c)-\frac{3015}{4^{1006}}=\frac{3018}{4^{1006}}-\frac{3015}{4^{1006}}=\frac{3}{4^{1006}}$$
Đẳng thức xảy ra khi và chỉ khi $a=b=c=\frac{1}{4}$ và $xyz=1 \blacksquare$
Ngoài ra với giả thiết và cách làm tương tự ta có bài toán tương tự như sau:
Tìm GTNN của $\frac{1}{(1+x)^{2013}}+\frac{1}{(1+y)^{2013}}+\frac{1}{(1+z)^{2013}}$

Bài viết đã được chỉnh sửa nội dung bởi Ispectorgadget: 06-04-2012 - 17:59

►|| The aim of life is self-development. To realize one's nature perfectly - that is what each of us is here for. ™ ♫


#196
alex_hoang

alex_hoang

    Thượng úy

  • Hiệp sỹ
  • 1152 Bài viết
Bài 89:Cho các số thực dương $x,y,z$ thỏa mãn điều kiện : $xy + yz + zx = 3.$ Chứng minh bất đẳng thức $$\dfrac{{x + y + z}}{3}\,\,\,\, \ge \,\,\,\sqrt[{36}]{{\dfrac{{{x^4} + {y^4} + {z^4}}}{3}}}.$$
Đề thi thử ĐH THPT Đặng Thúc Hứa
alex_hoang


HẸN NGÀY TRỞ LẠI VMF THÂN MẾN

http://www.scribd.co...oi-Ban-Cung-The

#197
alex_hoang

alex_hoang

    Thượng úy

  • Hiệp sỹ
  • 1152 Bài viết
Bài 90:Cho$x,y,z>0$thỏa mãn$xy+yz+zx=3xyz$Chứng minh rằng
$$\frac{yz\sqrt{1+3{{x}^{2}}}}{y+3zx}+\frac{zx\sqrt{1+3{{y}^{2}}}}{z+3xy}+\frac{xy\sqrt{1+3{{z}^{2}}}}{x+3yz}\ge \frac{3}{2}$$
alex_hoang


HẸN NGÀY TRỞ LẠI VMF THÂN MẾN

http://www.scribd.co...oi-Ban-Cung-The

#198
Ispectorgadget

Ispectorgadget

    Nothing

  • Quản lý Toán Phổ thông
  • 2946 Bài viết
Bài 91: Cho x,y,z dương thoả mãn $x+y+z=3$.Tìm min:
P=$\frac{x^2+3y^2}{xy^2.(4-xy)}+\frac{y^2+3z^2}{yz^2.(4-yz)}+\frac{z^2+3x^2}{zx^2.(4-xz)}$

(Đề thi thử lần II Quốc hoc Huế)

Bài 92:
Cho các số dương a,b,c thỏa mãn điều kiện a.b.c=1
Hãy tìm giá trị nhỏ nhất của biểu thức

P=$\frac{a}{\sqrt{a+1}} +\frac{b}{\sqrt{b+1}} + \frac{c}{\sqrt{c+1}}$

Đề thi thử THPT Yên Lãng.

►|| The aim of life is self-development. To realize one's nature perfectly - that is what each of us is here for. ™ ♫


#199
phuongnamz10A2

phuongnamz10A2

    Trung sĩ

  • Thành viên
  • 171 Bài viết

Bài 87:Cho $x,\,y,\,z$ là các số thực dương thỏa mãn $xyz=x+y+z+2.$ Chứng minh rằng $$\sqrt{x}+\sqrt{y}+\sqrt{z} \le \frac{3}{2}\sqrt{xyz}.$$

Trích đề thi trường Trần Quốc Tuấn lần 3 2012


Điểm rơi của BĐT là $x=y=z=2$
Ta chứng minh BĐT phụ sau.$$a+b+c+\sqrt{2abc}+2 \geq 2(\sqrt{ab}+\sqrt{bc}+\sqrt{ca})$$
Dễ có: $$2\sqrt{c}(\sqrt{a}-\sqrt{2})(\sqrt{b}-\sqrt{2}) \geq 0$$
và $$(\sqrt{a}-\sqrt{b})^2+(\sqrt{c}-\sqrt{2})^2 \geq 0$$
Khai triển 2 BĐT trên và cộng vế với vế ta có:
$$a+b+c+\sqrt{2abc}+2 \geq 2(\sqrt{ab}+\sqrt{bc}+\sqrt{ca})$$ (1)
Trở lại bài toán
$\sqrt{x}+\sqrt{y}+\sqrt{z} \le \frac{3}{2}\sqrt{xyz}.$
$<=> x+y+z+2(\sqrt{xy}+\sqrt{xz}+\sqrt{yz}) \le \frac{9}{4}xyz$
$<=> x+y+z+x+y+z+2+\sqrt{2xyz} \le \frac{9}{4}xyz$ (sử dụng BĐT 1)
$<=> \frac{1}{4}xyz+\sqrt{2xyz}-2 \le \frac{9}{4}xyz$
$<=> (\sqrt{xyz}-2\sqrt{2})^2 \geq 0 $ (luôn đúng)
Vậy BĐT được chứng minh.Dấu ''='' xảy ra khi $x=y=z=2$

Bài viết đã được chỉnh sửa nội dung bởi phuongnamz10A2: 17-05-2012 - 23:18


#200
Ispectorgadget

Ispectorgadget

    Nothing

  • Quản lý Toán Phổ thông
  • 2946 Bài viết
Bài 93: Cho $a,b,c$ thực dương thỏa $ab+bc+ac=12$. CMR:
$$\frac{1}{8+a^2(b+c)}+\frac{1}{8+b^2(a+c)}+\frac{1}{8+c^2(b+a)}\leq \frac{1}{abc}$$

Chuyên Lương Thế Vinh Đồng Nai

Bài viết đã được chỉnh sửa nội dung bởi NGOCTIEN_A1_DQH: 26-05-2012 - 21:36

►|| The aim of life is self-development. To realize one's nature perfectly - that is what each of us is here for. ™ ♫





0 người đang xem chủ đề

0 thành viên, 0 khách, 0 thành viên ẩn danh